[Date Prev][Date Next][Thread Prev][Thread Next][Date Index][Thread Index]

Re: [obm-l] E-mail do Tengan sobre o IMO 6



Legal,esta ideia e parecida com a minha.Mas uma
coisa:alguem pode ser mais explicito nesta parte
de olhar a raiz primitiva de q?E como e que a
ordem e exatamente p?

 --- edmilson motta <edeale@yahoo.com> escreveu:
> Ei pessoal,
> 
> voces notaram que o problema 6 da prova e' uma
> versao simplificada de um problema que eu e o
> Ed
> mandamos em uma das listas de treinamento do
> ano
> passado?  O problema da lista era algo assim:
> 
> Sejam a,r>1 e p um primo.  Prove que existe um
> primo q tal que (a mod q) tem ordem p^r.
> 
> Este e' o famoso lema de van der Waerden, que
> e'
> utilizado na prova do teorema de reciprocidade
> geral de Artin (mais detalhes, veja por exemplo
> Lang, Algebraic Number Theory, pag. 200).
> 
> A minha solucao e' curta demais pra um problema
> 6 da
> IMO,
> entao gostaria de pedir que voces checassem a 
> solucao.  Para nao irritar aqueles que ainda
> nao pensaram no problema, vou deixar um espaco
> em branco:
> 
> 
> 
> 
> 
> 
> 
> 
> 
> 
> 
> 
> 
> 
> 
> 
> 
> 
> 
> 
> 
> 
> 
> 
> 
> 
> 
> 
> 
> 
> 
> 
> 
> 
> 
> 
> 
> 
> 
> 
> 
> 
> 
> 
> 
> 
> 
> mais em baixo...
> 
> 
> 
> 
> 
> 
> 
> 
> 
> 
> 
> 
> 
> 
> 
> 
> 
> 
> 
> 
> 
> 
> 
> 
> 
> 
> 
> 
> 
> 
> 
> 
> 
> 
> 
> 
> 
> 
> 
> 
> 
> 
> 
> 
> 
> 
> 
> 
> 
> 
> 
> 
> 
> 
> 
> 
> 
> 
> 
> 
> 
> 
> 
> 
> 
> 
> 
> 
> mais um pouco...
> 
> 
> 
> 
> 
> 
> 
> 
> 
> 
> 
> 
> 
> 
> 
> 
> 
> 
> 
> 
> 
> 
> 
> 
> 
> 
> 
> 
> 
> 
> 
> 
> 
> 
> 
> 
> 
> 
> 
> 
> 
> 
> 
> 
> 
> 
> 
> ta' chegando...
> 
> 
> 
> 
> 
> 
> 
> 
> 
> 
> 
> 
> 
> 
> 
> 
> 
> 
> 
> 
> 
> 
> 
> 
> 
> 
> 
> 
> 
> 
> 
> 
> Agora sim, vamos ao problema.  Em primeiro
> lugar,
> olhando para uma raiz
> primitiva 
> de q, e' facil reduzir o problema a
> provar que existe um primo q tal que
> p mod q nao e' uma p-esima potencia, i.e.,
> p^{(q-1)/p} mod q nao e' 1 mod q.
> 
> Considere
> 
> N = (p^p-1)/(p-1) = p^(p-1) + ... + p + 1
> 
> Se q e' um primo que divide N e p-1, entao de
> N=p mod q, segue q=p, absurdo.  Entao para
> todo primo q que divide N, p mod q tem ordem
> exatamente p.  O problema acaba se p^2 nao
> divide q-1, mas se todos os primos que dividem
> N sao = 1 mod p^2, entao N = 1 mod p^2, o
> que e' um absurdo.
> 
> Agora vejam: se no lema de van der Waerden
> a=p, r=1, este e' exatamente o problema da IMO,
> com algumas pequenas modificacoes!  A solucao
> do problema da lista e' igualzinho `a
> demonstracao acima.  Eu lembro que o Alex e o
> Issao fizeram este problema, e acho que mais
> alunos tambem acertaram.  Espero que os
> pokemons tenham se lembrado do problema
> durante a prova!
> 
> Estamos melhorando: um problema na IMO e uma
> previsao acertada!  Alguem arrisca os proximos
> numeros da loto?
> 
> Ate'
> ET
> 
> 
> 
> __________________________________
> Do you Yahoo!?
> SBC Yahoo! DSL - Now only $29.95 per month!
> http://sbc.yahoo.com
>
=========================================================================
> Instruções para entrar na lista, sair da lista
> e usar a lista em
>
http://www.mat.puc-rio.br/~nicolau/olimp/obm-l.html
>
========================================================================= 

_______________________________________________________________________
Yahoo! Mail
Mais espaço, mais segurança e gratuito: caixa postal de 6MB, antivírus, proteção contra spam.
http://br.mail.yahoo.com/
=========================================================================
Instruções para entrar na lista, sair da lista e usar a lista em
http://www.mat.puc-rio.br/~nicolau/olimp/obm-l.html
=========================================================================